Diễn Đàn MathScopeDiễn Đàn MathScope
  Diễn Đàn MathScope
Ghi Danh Hỏi/Ðáp Thành Viên Social Groups Lịch Ðánh Dấu Ðã Ðọc

Go Back   Diễn Đàn MathScope > Sơ Cấp > Giải Tích > Các Bài Toán Đã Được Giải

News & Announcements

Ngoài một số quy định đã được nêu trong phần Quy định của Ghi Danh , mọi người tranh thủ bỏ ra 5 phút để đọc thêm một số Quy định sau để khỏi bị treo nick ở MathScope nhé !

* Nội quy MathScope.Org

* Một số quy định chung !

* Quy định về việc viết bài trong diễn đàn MathScope

* Nếu bạn muốn gia nhập đội ngũ BQT thì vui lòng tham gia tại đây

* Những câu hỏi thường gặp

* Về việc viết bài trong Box Đại học và Sau đại học


Trả lời Gởi Ðề Tài Mới
 
Ðiều Chỉnh Xếp Bài
Old 10-06-2011, 05:59 PM   #1
traidaohoa
+Thành Viên+
 
traidaohoa's Avatar
 
Tham gia ngày: Dec 2010
Đến từ: Hà Tây quê lụa
Bài gởi: 9
Thanks: 9
Thanked 0 Times in 0 Posts
Gửi tin nhắn qua Yahoo chát tới traidaohoa
Tìm max trên hình vuông.

Cho hàm số $ f0,1] \times [0,1] \rightarrow R $ bởi $f(x,y) =\left\{\begin{matrix}
y(1-x) if y \leq x &\\
x(1-y) if y>x
\end{matrix}\right. $
Tìm giá trị lớn nhất của f trên hình vuông $ [0,1] \times [0,1]. $
[RIGHT][I][B]Nguồn: MathScope.ORG[/B][/I][/RIGHT]
 
__________________
...
Cho tôi một vé đi tuổi thơ
...
traidaohoa is offline   Trả Lời Với Trích Dẫn
Old 10-06-2011, 06:37 PM   #2
Huy_92
+Thành Viên+
 
Tham gia ngày: Apr 2010
Đến từ: Đại học Bách khoa Hà nội
Bài gởi: 439
Thanks: 94
Thanked 215 Times in 136 Posts
Chào bạn,

Hướng làm của bài toán này như sau:

Nếu là hàm không điều kiện trên tập giới hạn biên

1.Giải phương trình các đạo hàm riêng để tìm các điểm dừng

2.Tính giá trị của hàm tại các điểm dừng (*)

3.Tìm giá trị lớn nhất của hàm trên biên

4.So sánh với các giá trị (*) và kết luận

Còn với các bài toán có điều kiện ta sử dụng phép toán lagrange nhiều cuốn giáo trình có viết

Thời gian này hầu hết mọi người đều ôn thi cuối kì,nên hơi ít thời gian.Bạn cố gắng giải quyết vấn đề theo gợi ý nhé
[RIGHT][I][B]Nguồn: MathScope.ORG[/B][/I][/RIGHT]
 

thay đổi nội dung bởi: Huy_92, 10-06-2011 lúc 08:50 PM
Huy_92 is offline   Trả Lời Với Trích Dẫn
Old 10-06-2011, 08:19 PM   #3
leviethai
+Thành Viên+
 
Tham gia ngày: Nov 2008
Đến từ: Thành phố Hồ Chí Minh. Nhưng quê tôi là Ninh Bình.
Bài gởi: 513
Thanks: 121
Thanked 787 Times in 349 Posts
Gửi tin nhắn qua Yahoo chát tới leviethai
Bài này em thấy hình như đơn giản mà.

Khi mà $y\le x $ thì
$f(x,y)=y(1-x)\le x(1-x) \le \dfrac{1}{4} $ . (áp dụng AM-GM)

Khi mà $x<y $ thì $f(x,y)<\dfrac{1}{4}. $

Khi $x=y=\dfrac{1}{2} $ thì $f(x,y)=\dfrac{1}{4} $.

Vậy giá trị lớn nhất của $f(x,y) $ là $\dfrac{1}{4} $. $\hfill \Box $

Không biết có hiểu nhầm chỗ nào không nhỉ ?

[RIGHT][I][B]Nguồn: MathScope.ORG[/B][/I][/RIGHT]
 
leviethai is offline   Trả Lời Với Trích Dẫn
The Following User Says Thank You to leviethai For This Useful Post:
sythanh14 (23-04-2012)
Trả lời Gởi Ðề Tài Mới

Bookmarks

Ðiều Chỉnh
Xếp Bài

Quuyền Hạn Của Bạn
You may not post new threads
You may not post replies
You may not post attachments
You may not edit your posts

BB code is Mở
Smilies đang Mở
[IMG] đang Mở
HTML đang Tắt

Chuyển đến


Múi giờ GMT. Hiện tại là 03:51 PM.


Powered by: vBulletin Copyright ©2000-2024, Jelsoft Enterprises Ltd.
Inactive Reminders By mathscope.org
[page compression: 44.94 k/49.62 k (9.41%)]